How can the time taken for a loop to roll down an incline be calculated?

In summary, the homework statement states that the force on the loop due to magnetic field in the direction normal to the incline is ##qvB##. The Attempt at a Solution states that the balancing forces in the direction normal to the incline, mg\cos\alpha=qvB+N, when the loop takes off, N=0 i.e mg\cos\alpha=qvB. Also, ##v=g\sin\alpha t##. Substituting this in the above expression and solving for t, I get ##t=1 sec## but this is wrong.
  • #1
Saitama
4,243
93

Homework Statement


attachment.php?attachmentid=59829&stc=1&d=1372160756.png

Homework Equations


The Attempt at a Solution


The force on the loop due to magnetic field in the direction normal to the incline is ##qvB##.
Balancing forces in the direction normal to the incline,
[tex]mg\cos\alpha=qvB+N[/tex]
where ##N## is the normal reaction due to incline and v is the velocity of CM of loop.
When the loop takes off, N=0 i.e
[tex]mg\cos\alpha=qvB[/tex]
Also, ##v=g\sin\alpha t##. Substituting this in the above expression and solving for t, I get ##t=1 sec## but this is wrong. :confused:
 

Attachments

  • LOOP.png
    LOOP.png
    19.6 KB · Views: 439
  • 3829-small.jpg
    3829-small.jpg
    13.2 KB · Views: 367
Physics news on Phys.org
  • #2
Hi Pranav-Arora! :smile:
Pranav-Arora said:
Also, ##v=g\sin\alpha t##.

Nope. :redface:

If you got this from F = ma, you left out the static friction force. :wink:

(were you really so optimistic as to think that you could solve this without using the rolling constratint? :rolleyes:)
 
  • #3
I would say if the loop has a mass, you will have to account for its rolling motion. This means use the rotational inertia for a thin loop!
 
  • #4
tiny-tim said:
Nope. :redface:

If you got this from F = ma, you left out the static friction force. :wink:
Ah yes, completely missed it. But how am I supposed to calculate the friction force here? Do I have to go like this:
##mg\sin\alpha-f_R=ma## (Translatory motion)
##f_RR=I\beta=mR^2a/R## (Torque about CM, ##\beta## is the angular acceleration)
Solving the two equations:
##a=g\sin\alpha/2##
Hence, ##v=g\sin\alpha t/2##
I had,
##mg\cos\alpha=qvB##
Substituting v and solving for t, ##t=2 sec##, looks correct?
 
  • #5
I think what tiny-tim meant was that you forgot about an effect of static friction. You should not have to calculate any friction directly in this problem. But, what the static friction does, is it causes the loop to roll, not slide. If there was no static friction, the loop would slide instead of roll, causing it to have 0 angular momentum. Since there is static friction though (implicitly implied), you must take into the moment of inertia of the loop.
 
  • #6
spaderdabomb said:
I think what tiny-tim meant was that you forgot about an effect of static friction. You should not have to calculate any friction directly in this problem. But, what the static friction does, is it causes the loop to roll, not slide. If there was no static friction, the loop would slide instead of roll, causing it to have 0 angular momentum. Since there is static friction though (implicitly implied), you must take into the moment of inertia of the loop.

I did take moment of inertia into account. Isn't my last post clear enough? :redface:
 
  • #7
Ah ok missed it sorry
 
  • #8
Hmmm I don't like what you did still though. There shouldn't be any friction in this problem. Is there any sliding occurring? If yes, then there should be friction in this problem. If no, then there should not be any friction. It only has the efffect of causing rotation. The frictional force you put in there kind of came out of nowhere (unless you can somehow justify it to me?)
 
  • #9
Actually I am sorry, you did it fine. I just thought it was weird how you put the friction in there to begin with. I would have gone straight to the moment of inertia haha.

Also I should add, using forces may not be the best method. It will be much easier to use energy. We can assign a given height 'h' where the hoop takes off. If we call ground 0 potential energy, then energy due to gravity is -mgh. Our kinetic energy of the ball moving hoop is 1/2mv^2 and the energy from the rotations of the hoop is 1/2Iω^2 where ω is the angular velocity, a function of velocity (ω=v/r). Now use conservation of energy. But if what you did gets you the correct answer then that is fine.
 
  • #10
Hi Pranav-Arora! :smile:

(sorry, I've been out, enjoying myself! :biggrin:)
Pranav-Arora said:
Ah yes, completely missed it. But how am I supposed to calculate the friction force here? Do I have to go like this:
##mg\sin\alpha-f_R=ma## (Translatory motion)
##f_RR=I\beta=mR^2a/R## (Torque about CM, ##\beta## is the angular acceleration)
Solving the two equations:
##a=g\sin\alpha/2##
Hence, ##v=g\sin\alpha t/2##

yes that's fine :smile:

btw, you can check it by using the "rolling mass", equal to I/r2, in this case m (for a disc it's m/2, for a sphere it's 2m/5)

you add the "rolling mass" to actual mass giving 2m, then you use that in F = ma

(so for a disc v = gsinαt/(m + m/2) = 2gsinαt/3, and for a sphere = 5gsinαt/7) …

but i don't think the examiners would like that, so just use it for checking :wink:

you can also find v by τ = Iα about the point of contact (since it's the centre of rotation, that's ok), which is quicker since it avoids using (and eliminating) the static friction force :wink:
 
  • #11
Thank you for the help tiny-tim! :smile:

tiny-tim said:
Hi Pranav-Arora! :smile:

(sorry, I've been out, enjoying myself! :biggrin:)


yes that's fine :smile:

btw, you can check it by using the "rolling mass", equal to I/r2, in this case m (for a disc it's m/2, for a sphere it's 2m/5)

you add the "rolling mass" to actual mass giving 2m, then you use that in F = ma

(so for a disc v = gsinαt/(m + m/2) = 2gsinαt/3, and for a sphere = 5gsinαt/7) …

but i don't think the examiners would like that, so just use it for checking :wink:

you can also find v by τ = Iα about the point of contact (since it's the centre of rotation, that's ok), which is quicker since it avoids using (and eliminating) the static friction force :wink:

Great! Thank you for both the tips, these will surely be helpful ahead. :)
 

Related to How can the time taken for a loop to roll down an incline be calculated?

1. What is loop rolling on an incline?

Loop rolling on an incline is a physics phenomenon where a small object, such as a marble, is placed on an inclined plane and is able to loop multiple times before coming to a stop.

2. Why does loop rolling occur on an incline?

Loop rolling occurs on an incline due to the combination of gravity and the shape of the incline. The incline's curve provides a centripetal force that allows the object to loop without falling off the incline, while gravity provides the downward force that keeps the object moving.

3. What factors affect loop rolling on an incline?

The factors that affect loop rolling on an incline include the angle of the incline, the shape and smoothness of the incline, the mass and size of the object, and the force of gravity. These factors can impact the speed, number of loops, and stability of the rolling object.

4. How is loop rolling on an incline related to other physics concepts?

Loop rolling on an incline is related to concepts such as centripetal force, gravity, and potential and kinetic energy. The loop rolling motion can also be analyzed using equations such as Newton's Second Law and conservation of energy.

5. What are some real-life applications of loop rolling on an incline?

Loop rolling on an incline can be seen in various amusement park rides, such as roller coasters and water slides. It can also be used in engineering and design to understand the motion of objects on curved surfaces, such as cars on a banked turn or balls in a curved race track.

Similar threads

  • Introductory Physics Homework Help
3
Replies
97
Views
3K
Replies
24
Views
1K
  • Introductory Physics Homework Help
Replies
12
Views
318
Replies
10
Views
582
  • Introductory Physics Homework Help
Replies
8
Views
3K
  • Introductory Physics Homework Help
Replies
18
Views
3K
  • Introductory Physics Homework Help
Replies
2
Views
933
  • Introductory Physics Homework Help
Replies
2
Views
377
  • Introductory Physics Homework Help
Replies
5
Views
1K
  • Introductory Physics Homework Help
Replies
2
Views
891
Back
Top